股票

複製具有特定收益函式的投資組合

  • July 13, 2017

假設有兩隻股票 $ S_1 $ 有價格 $ p_1(t) $ 和 $ S_2 $ 有價格 $ p_2(t) $ 在哪裡 $ t $ 表示時間。假設有一個假設導數 $ D $ ,也就是說,價格 $ D $ 一次 $ t $ 是(誰)給的 $ p_1(t)/p_2(t) $ .

是否有可能找到使用股票和債券來建構這種衍生品的動態對沖策略?

如果是,那麼如何?

一般對沖策略

假設 $ S_1(t) $ 和 $ S_2(t) $ 是您的 2 隻股票的價格過程,它們遵循幾何布朗運動 (GBM):

$$ \forall , i \in {1,2}, dS_i(t) =\mu_iS_i(t)dt + \sigma_iS_i(t)dW_i(t) $$ 我們假設兩隻股票的即時相關性為 $ \rho $ :

$$ dW_1(t)dW_2(t)=\rho dt $$ 也讓 $ V(t) $ 是取決於價格的衍生品的價值或(公平)價格 $ S_1(t) $ 和 $ S_2(t) $ 有時 $ t $ . 我們建構了一個自籌資金的投資組合,由 $ w_0(t) $ 衍生合約, $ w_1(t) $ 股票 $ 1 $ 和 $ w_2(t) $ 股票 $ 2 $ . 它的價值在 $ t $ , $ \Pi(t) $ , 是(誰)給的:

$$ \Pi(t) = w_0(t)V(t)+w_1(t)S_1(t)+w_2(t)S_2(t) $$ 自籌資金的投資組合價值根據以下因素演變:

$$ d\Pi(t) = w_0(t)dV(t) + w_1(t)dS_1(t) + w_2(t)dS_2(t) $$ 然後我們有 $ - $ 掉落時間:

$$ dV = \frac{\partial V}{\partial t}dt + \frac{\partial V}{\partial S_1}dS_1 + \frac{\partial V}{\partial S_2}dS_2 + \frac{1}{2}\frac{\partial^2 V}{\partial S_1^2}dS_1^2 + \frac{1}{2}\frac{\partial^2 V}{\partial S_2^2}dS_2^2 + \frac{\partial^2 V}{\partial S_1 \partial S_2}dS_1dS_2 $$ 在上述微分方程中,乘以權重 $ w_0(t) $ ,隨機元素為:

$$ \frac{\partial V}{\partial S_1}w_0\sigma_1S_1dW_1 + \frac{\partial V}{\partial S_2}w_0\sigma_2S_2dW_2 $$ 現在,在 $ w_1(t)S_1(t)+w_2(t)S_2(t) $ ,隨機元素為:

$$ w_1\sigma_1S_1dW_1 + w_2\sigma_2S_2dW_2 $$ 我們正在對沖衍生品 $ V(t) $ ,因此我們的投資組合必須是無風險的並獲得無風險利率

  • 從無風險的條件下,必須消除隨機波動。使用投資組合價值方程,我們推導出對沖策略:

$$ \begin{align} w_0(t) & = \frac{\Pi(t)}{V(t)-S_1(t)\frac{\partial V}{\partial S_1}-S_2(t)\frac{\partial V}{\partial S_2}} \[12pt] w_1(t) &= -w_0(t)\frac{\partial V}{\partial S_1} \[12pt] w_2(t) &= -w_0(t)\frac{\partial V}{\partial S_2} \end{align} $$

  • 我們讓 $ B(t) = e^{rt} $ 成為無風險債券,賺取無風險利率 $ r $ . 鑑於投資組合必須賺取 $ r $ 並假設 $ \Pi(0) $ 被正規化以便等於 $ 1 $ , 然後 $ B(t) $ 是無風險收益約束的解:

$$ d\Pi(t) = r\Pi(t) dt $$ 對沖組合的最終表達式為:

$$ \begin{align} w_0(t) & = \frac{B(t)}{V(t)-S_1(t)\frac{\partial V}{\partial S_1}-S_2(t)\frac{\partial V}{\partial S_2}} \[12pt] w_1(t) &= -w_0(t)\frac{\partial V}{\partial S_1} \[12pt] w_2(t) &= -w_0(t)\frac{\partial V}{\partial S_2} \end{align} $$ 你的問題

現在,在您的衍生產品的特殊情況下,並嚴格解釋您的原始問題:

"

$$ … $$ 價格 $$ the derivative $$ $ D $ 一次 $ t $ 是(誰)給的 $ \frac{S_1(t)}{S_2(t)} $ . "

然後價格由下式給出:

$$ V(t) = \frac{S_1(t)}{S_2(t)} $$ 我們有 $ \partial V/\partial S_1 = 1/S_2 = V/S_1 $ 和 $ \partial V/\partial S_2 = -S_1/S_2^2 = -V/S_2 $ ,因此對沖策略是:

$$ \begin{align} w_0(t) &= \frac{B(t)}{V(t)} \[12pt] w_1(t) &= -\frac{B(t)}{V(t)}\frac{\partial V}{\partial S_1} = -w_0(t)\frac{\partial V}{\partial S_1} \[12pt] w_2(t) &= -\frac{B(t)}{V(t)}\frac{\partial V}{\partial S_2} = -w_0(t)\frac{\partial V}{\partial S_2} \end{align} $$ 但是,我不確定定價 $ V(t) $ 是正確的方法:衍生品的價格為 $ t $ 應該從它的支付函式和無風險回報條件產生的 PDE 推導出來。經過幾個步驟,我們將得到以下 PDE $ - $ 掉落時間:

$$ \begin{align} rV & = \frac{\partial V}{dt} + \frac{\partial V}{\partial S_1}rS_1 + \frac{1}{2}\frac{\partial^2V}{\partial S_1^2}\sigma_1^2S_1^2 + \frac{\partial V}{\partial S_2}rS_2 + \frac{1}{2}\frac{\partial^2V}{\partial S_2^2}\sigma_2^2S_2^2 + \frac{\partial^2V}{\partial S_1 \partial S_2}\sigma_1\sigma_2S_1S_2\rho \[12pt] & = \frac{\partial V}{dt} + rw_1S_1 + \frac{1}{2}\frac{\partial^2V}{\partial S_1^2}\sigma_1^2S_1^2 + rw_2S_2 + \frac{1}{2}\frac{\partial^2V}{\partial S_2^2}\sigma_2^2S_2^2 + \frac{\partial^2V}{\partial S_1 \partial S_2}\sigma_1\sigma_2S_1S_2\rho \end{align} $$ 解決它將產生 $ V(t) $ . 注意重量 $ w_0 $ 沒有出現在上面的 PDE 中,因為所有三個權重 $ w_0 $ , $ w_1 $ 和 $ w_2 $ 可以寫成一個函式 $ w_0 $ ,因此術語 $ w_0 $ 最終被取消。

在這種特殊情況下,通過用它們的特定表達式替換導數 $ - $ 我們可以從以下事實中得出 $ V(t) = S_1(t)/S_2(t) $ $ - $ 我們獲得:

$$ \begin {align} & : rV = 0 + rV + 0 - rV + \sigma_2^2V - \sigma_1\sigma_2\rho V \[12pt] \Leftrightarrow & : r = \sigma_2^2 - \sigma_1\sigma_2\rho \end{align} $$ 因此,我們將對“市場”參數施加約束。正確的方法是推導出一個表達式 $ V(t) $ 給定上面的 PDE。

最後一個例子: $ V(T)=\frac{S_1(T)}{S_2(T)} $

假設您的衍生品具有以下收益函式:

$$ V_T=V(T)=\frac{S_1(T)}{S_2(T)} $$ 切換到馬丁格爾定價工具,我們知道:

$$ \forall , t \in [0,T], V_t = \mathbb{E}^{\mathbb{Q}}\left[e^{-r(T-t)}\frac{S_1(T)}{S_2(T)}|\mathcal{F}_t\right] $$ 在哪裡 $ \mathbb{Q} $ 是風險中性度量。現在讓我們定義 $ X_t = X(t) = S_1(t)/S_2(t) $ . 應用伊藤引理 $ - $ 和下降時間:

$$ \begin{align} dX_t = \frac{dS_1}{S_2} - \frac{S_1dS_2}{S_2^2} + \frac{S_1dS_2^2}{S_2^3} - \frac{dS_1dS_2}{S_2^2} \[12pt] \Leftrightarrow \frac{dX_t}{X_t} = \left(\mu_1-\mu_2-\sigma_1\sigma_2\rho+\sigma_2^2\right)dt +\sigma_1dW_1^{\mathbb{Q}} + \sigma_2dW_2^{\mathbb{Q}} \end{align} $$ 在風險中性措施下, $ \mu_1=\mu_2=r $ . 讓 $ W_i(t) = W_i^{\mathbb{Q}}(t) $ ,經過幾步我們得到:

$$ \begin{align} X_T = X_te^{(\sigma_2^2-\sigma_1^2-4\sigma_1\sigma_2\rho)\frac{T-t}{2}+\sigma_1W_1(T-t)+\sigma_2W_2(T-t)} \end{align} $$ 將伊藤引理應用於 $ W_1(t)W_2(t) $ 並使用 Ito 積分的鞅性質,我們得到:

$$ \mathbb{Cov}[W_1(T-t),W_2(T-t)] = \rho (T-t) $$ 現在,讓:

$$ \begin{align} & Z(t) = (\sigma_2^2-\sigma_1^2-4\sigma_1\sigma_2\rho)\frac{t}{2}+\sigma_1W_1(t)+\sigma_2W_2(t) \[6pt] & \mathbb{E}[Z(t)] = (\sigma_2^2-\sigma_1^2-4\sigma_1\sigma_2\rho)\frac{t}{2} \[6pt] & \mathbb{V}[Z(t)] = \sigma_1^2t + \sigma_2^2t+2\sigma_1\sigma_2\rho t \end{align} $$ 我們獲得:

$$ \mathbb{E}[X_T|\mathcal{F}_t] = X_te^{\sigma_2^2(T-t)-\sigma_1\sigma_2\rho (T-t)} $$ 因此

$$ \forall , t \in [0,T], V_t = \frac{S_1(t)}{S_2(t)}e^{(\sigma_2^2-\sigma_1\sigma_2\rho-r)(T-t)} $$ 我們回到了“您的問題”部分中對市場參數的約束:

$$ r = \sigma_2^2 - \sigma_1\sigma_2\rho \quad \Rightarrow \quad V_t = \frac{S_1(t)}{S_2(t)} $$ 對沖策略將是:

$$ \begin{align} & w_0(t) = \frac{S_2(t)}{S_1(t)}e^{(\sigma_1\sigma_2\rho-\sigma_2^2)(T-t)+rT} \[12pt] & w_1(t) = -\frac{e^{rt}}{S_1(t)} \[12pt] & w_2(t) = \frac{e^{rt}}{S_2(t)} \end{align} $$ 您可以輕鬆檢查 $ w_0(t)V(t)+w_1(t)S_1(t)+w_2(t)S_2(t) = B(t) $ .

我們終於可以檢查了:

$$ \begin{align} & \frac{\partial V}{dt} = -(\sigma_2^2-\sigma_1\sigma_2\rho-r)V \[12pt] & \frac{\partial V}{\partial S_1}rS_1 = rV \[12pt] & \frac{1}{2}\frac{\partial^2V}{\partial S_1^2}\sigma_1^2S_1^2 = 0 \[12pt] & \frac{\partial V}{\partial S_2}rS_2 = -rV \[12pt] & \frac{1}{2}\frac{\partial^2V}{\partial S_2^2}\sigma_2^2S_2^2 = \sigma_2^2V \[12pt] & \frac{\partial^2V}{\partial S_1 \partial S_2}\sigma_1\sigma_2S_1S_2\rho = -\sigma_1\sigma_2\rho V \end{align} $$ 項取消,我們得到以下恆等式:

$$ rV = rV $$ 因此 $ V(t) $ 如上所述是定價 PDE 的解決方案。

更多參考資料

來自 Wikipedia 和使用者 @Gordon 的有關衍生工具的套期保值策略和定價 PDE 的更多相關參考資料 $ V(t) $ :

引用自:https://quant.stackexchange.com/questions/34414